- Thu Aug 30, 2018 4:07 pm
#55812
Complete Question Explanation
(The complete setup for this game can be found here: lsat/viewtopic.php?t=15077)
The correct answer choice is (B)
This is an unusually easy question. The first rule of the game establishes that Z must be purchased by the museum. Thus, any answer choice in this question featuring Z must be incorrect. Using that logic, we can eliminate answer choices (A), (D), and (E).
The question stem places S in the artist’s second period, which, from our initial discussion of the rules, forces Z into the artist’s third period. V, which must also be from the artist’s second period, has to be purchased by the museum:
At this point, we have established that V must be purchased by the museum, and consequently answer choice (C) can be eliminated.
Answer choice (B): This is the correct answer choice.
(The complete setup for this game can be found here: lsat/viewtopic.php?t=15077)
The correct answer choice is (B)
This is an unusually easy question. The first rule of the game establishes that Z must be purchased by the museum. Thus, any answer choice in this question featuring Z must be incorrect. Using that logic, we can eliminate answer choices (A), (D), and (E).
The question stem places S in the artist’s second period, which, from our initial discussion of the rules, forces Z into the artist’s third period. V, which must also be from the artist’s second period, has to be purchased by the museum:
At this point, we have established that V must be purchased by the museum, and consequently answer choice (C) can be eliminated.
Answer choice (B): This is the correct answer choice.
You do not have the required permissions to view the files attached to this post.